2
$\begingroup$

Let $G$ be a profinite group which fits into the following short exact sequence: $$ 1\rightarrow N\rightarrow G \rightarrow K\rightarrow 1 $$ Assume that $N$ is a pro-$p$ group and that $K$ is topologically finitely generated. Note that $K$ acts naturally by conjugation on $N^{ab}$ and thus we way view $N^{ab}$ as a $\mathbf{Z}_p[[K]]$-module.

Q: If $N^{ab}$ is a finite type $\mathbf{Z}_p[[K]]$-module, does it follow that $G$ is topologically finitely generated?

$\endgroup$
6
  • 2
    $\begingroup$ Could you please explain what finite type means? Generally, $N$ is generated as a normal subgroup of $G$ by a set $X$ if and only if $N/([N,G]N^p)$ is generated by the image of $X$. So if $N^{ab}$ is a finitely generated $\mathbb{Z}_p[[K]]$-module, then the answer to your question should be true. $\endgroup$ Jun 3, 2012 at 7:32
  • $\begingroup$ @Yiftach "finite type" means finitely generated (by a word-by-word translation from French). So you can put your comment as an answer. $\endgroup$
    – YCor
    Jun 3, 2012 at 7:59
  • $\begingroup$ @Yiftach, I think I understand how to do it when the groups are finite. So let $K=<x_1,\ldots,x_r>$ and let $\{n_1,\ldots,n_s\}$ be the generators of the $\mathbf{Z}_p[[K]]$-module. Then by what you said one has that the normal closure of $<x_i*n_j: i,j>$ which I denote by $NC<x_i*n_j: i,j>$ generated $N$. Now if $<x_i*n_j: i,j>$ was contained in a maximal subgroup $M$ of $N$ then because $N$ is a $p$-group we know that $M$ is normal and thus we would have $NC<x_i*n_j: i,j>\subseteq M$ which is a contradiction. $\endgroup$ Jun 3, 2012 at 14:18
  • $\begingroup$ @Hugo: the profinite case follows from the finite case (a subgroup is dense iff all its images in finite quotients are surjective; apply this to the normal subgroup generated by a finite subset topologically generating $N$ modulo the commutators). $\endgroup$
    – YCor
    Jun 3, 2012 at 14:25
  • $\begingroup$ PS: by normal here I mean normal in $G$ $\endgroup$
    – YCor
    Jun 3, 2012 at 14:26

1 Answer 1

3
$\begingroup$

So here is my comment again with slightly more details. Let $Y$ be a finite subset of $G$ such that its image generates $K$. As I was told by Yves, finite type means finitely generated. Thus, let $X$ be a finite subset of $N$ such that $N^{ab}$ is generated by its image as a $\mathbb{Z}_p[[K]]$-module. Then we claim that $H$ the subgroup generated (topologically) by $X \cup Y$ is equal $G$. Since $Y \subseteq H$ we have that $HN/N \cong K$.

Therefore, $HN=G$. So it suffices to show that $N \leq H$. By definition, the action of $K$ on $N^{ab}$ is via the action of $G$. Since $N$ acts trivially on $N^{ab}$ we have that $H$ action on $N^{ab}$ is the same as $HN=G$ action. Hence, the image of $Y^{H}$ generates $N^{ab}$ as a $\mathbb{Z_p}$-module. So $Y^{H}$ generates $N/\Phi(N)$ as a group, where $\Phi(N)=[N,N]N^{p}$ is the Frattini subgroup of $N$. We deduce that $Y^{H}$ generates $N$ (topologically) and $N \leq H$ as we wanted.

Note that it suffices to ask that $N/([N,N]N^{p})$ (EDIT: see comments below) is finitely generated as a $K$-module.

$\endgroup$
12
  • $\begingroup$ I had to break the first paragraph in a strange place since the latx got crazy. $\endgroup$ Jun 3, 2012 at 18:01
  • $\begingroup$ at the end you mean $N/([N,N]N^p)$, not $N/([N,K]N^p)$. $\endgroup$
    – YCor
    Jun 3, 2012 at 19:44
  • $\begingroup$ Yves, you are right. I was trying to be too clever, $[N,K]$ does not necessarily has a meaning as $K$ does not necessarily act on $N$. I think it is possible to replace it by $[N,G]$, but I have to check it to be sure. So I'll leave this to the reader and I will fix my mistake. $\endgroup$ Jun 3, 2012 at 22:02
  • $\begingroup$ @Yiftach: no you can't, even for a split extension. We can have $N=[N,K]$ then, but $N$ can be huge. For instance, pick $K$ to be cyclic on 2 elements, and $N$ an infinite direct power of a fixed cyclic group of odd order, $K$ acting by $x \mapsto -x$. $\endgroup$
    – YCor
    Jun 3, 2012 at 22:49
  • $\begingroup$ @Yves: again you are right. I am more confident that it is true if $G$ is pro-$p$. But it is too late to think about it now. $\endgroup$ Jun 3, 2012 at 23:13

Your Answer

By clicking “Post Your Answer”, you agree to our terms of service and acknowledge you have read our privacy policy.

Not the answer you're looking for? Browse other questions tagged or ask your own question.